WBR0012: Difference between revisions

Jump to navigation Jump to search
No edit summary
m (Gerald Chi moved page TestQues9 to WBR0012 without leaving a redirect)
(No difference)

Revision as of 23:59, 5 August 2013

 
Author PageAuthor::Aarti Narayan
Exam Type ExamType::USMLE Step 2 CK
Main Category MainCategory::Internal medicine
Sub Category SubCategory::Cardiovascular
Prompt [[Prompt::A 35 yr old woman presents to the out-patient department with symptoms of fatigue and dyspnea and pedal edema for the past 6 months. The symptoms were gradual in onset. On examination, she was found to have hepatomegaly and mild ascites. She had a past history of Rheumatic fever. Echocardiographic studies show thickening and stenosis of tricuspid valve leaflets. What is expected finding on jugular venous pressure tracing?]]
Answer A AnswerA::Absent ‘a’ wave
Answer A Explanation AnswerAExp::Absent ‘a’ wave is seen in patients with atrial fibrillation.
Answer B AnswerB::Deep ‘y’ descent
Answer B Explanation AnswerBExp::Deep ‘y’ descent is seen in early diastole with severe tricuspid regurgitation. It is also seen with constrictive pericarditis and severe right ventricular failure.
Answer C AnswerC::Giant ‘a’ wave
Answer C Explanation AnswerCExp::See Explanation
Answer D AnswerD::Prominent ‘v’ wave
Answer D Explanation AnswerDExp::Prominent ‘v’ wave results from increased volume of blood in right atrium during ventricular systole and is seen in tricuspid regurgitation. It is also seen with constrictive pericarditis and severe right ventricular failure.
Answer E AnswerE::Prominent ‘x’ descent
Answer E Explanation AnswerEExp::Prominent ‘x’ descent is seen in constrictive pericarditis.
Right Answer RightAnswer::C
Explanation [[Explanation::The correct answer is giant ‘a’ wave. The patient in this vignette has developed right sided heart failure secondary to tricuspid stenosis, a condition nearly always caused by rheumatism. Giant ‘a’ waves indicate that the right atrium is contracting against increased resistance. They are also seen in pulmonary hypertension and pulmonic stenosis. The image above demonstrates an example of a Giant ‘a’ wave in a patient with pulmonary hypertension secondary to cardiomyopathy. Note the absence of c and v waves in the JVP tracing.

Educational Objective:
References: ]]

Approved Approved::No
Keyword
Linked Question Linked::
Order in Linked Questions LinkedOrder::